MedSurg 2- Test 1

Pataasin ang iyong marka sa homework at exams ngayon gamit ang Quizwiz!

A nurse is working on a neurological unit with a nursing student who asks the difference between primary and secondary headaches. The nurse's correct response will include which of the following statements?

"A secondary headache is associated with an organic cause, such as a brain tumor."

A client is treated in the emergency department for acute muscle strain in the left leg caused by trying a new exercise. During discharge preparation, the nurse should provide which instruction?

"Apply ice packs for the first 24 to 48 hours, then apply heat packs."

A female patient has presented to the emergency department (ED) with complaints of a high fever and severe headache. The patient states that acetaminophen has had no appreciable effect on either symptom. The triage nurse recognizes the need to perform a rapid assessment for possible meningitis and should ask which of the following questions:

"Are you having stiffness or pain in your neck?"

The spouse of a client with terminal brain cancer asks the nurse about hospice. Which statement by the nurse best describes hospice care?

"Clients and families are the focus of hospice care."

The nurse is conducting the admission assessment for the client who is to undergo an arthrogram. What is the priority question the nurse should ask?

"Do you have any allergies?"

To evaluate a client's cerebellar function, a nurse should ask:

"Do you have any problems with balance?"

A nurse is instructing the spouse of a client who suffered a stroke about the use of eating devices the client will be using. During the teaching, the spouse starts to cry and states, "One minute he is laughing, and the next he's crying; I just don't understand what's wrong with him." Which statement is the best response by the nurse?

"Emotional lability is common after a stroke, and it usually improves with time."

A nurse is teaching a client who was recently diagnosed with carpal tunnel syndrome. Which statement should the nurse include?

"Ergonomic changes can be incorporated into your workday to reduce stress on your wrist."

The community health nurse is conducting a home visit with a client who was discharged from hospital 3 days ago after surgical resection of a brain tumor and radiation therapy. The client is accompanied by his partner during the nurse's visit. During the visit, the client's partner becomes tearful. How should the nurse respond?

"Going through this experience with your partner has been very difficult for you, I'm sure. Can you tell me about your experience so far?"

The nurse is caring for a client following a spinal cord injury who has a halo device in place. The client is preparing for discharge. Which statement by the client indicates the need for further instruction?

"I can apply powder under the liner to help with sweating."

A client undergoes a total hip replacement. Which statement made by the client indicates to the nurse that the client requires further teaching

"I don't know if I'll be able to get off that low toilet seat at home by myself."

A client undergoes an arthroscopy at the outpatient clinic. After the procedure, the nurse provides discharge teaching. Which response by the client indicates the need for further teaching?

"I should use my heating pad this evening to reduce some of the pain in my knee."

The nurse has educated a patient with low back pain about techniques to relieve the back pain and prevent further complications. What statement by the patient shows understanding of the education the nurse provided?

"I will avoid prolonged sitting or walking."

A client is being discharged from the Emergency Department after being diagnosed with a sprained ankle. Which client statement indicates the client understands the discharge teaching?

"I'll make sure to keep my ankle elevated as much as possible."

The nurse is preparing a client for a neurological examination by the physician and explains tests the physician will be doing, including the Romberg test. The client asks the purpose of this particular test. The correct reply by the nurse is which of the following?

"It is a test for balance."

A client has sustained a right tibial fracture and has just had a cast applied. Which instruction should the nurse provide in his cast care?

"Keep your right leg elevated above heart level."

A nurse on a neurological unit is participating in the care of a female patient who is receiving treatment for a spinal cord injury (SCI) that she experienced 2 weeks ago. The patient's care plan specifies measures to prevent skin breakdown, and the nurse has planned several changes of position during the shift. How should the nurse best reposition this patient?

"Log roll" the patient.

A client with newly diagnosed seizures asks about stigma associated with epilepsy. The nurse will respond with which of the following statements?

"Many people with developmental disabilities resulting from neurologic damage also have epilepsy."

A nurse is conducting a neurological assessment of a patient who has just been admitted to the unit. In preparation for assessing the patient for pronator drift, what instructions should the nurse provide to the patient?

"Please hold your arms straight out with your palms pointing up to the ceiling."

A nurse is preparing a client for lumbar puncture. The client has heard about post-lumbar puncture headaches and asks how to avoid having one. The nurse tells the client that these headaches can be avoided by doing which of the following after the procedure?

"Remain prone for 2 to 3 hours."

On a visit to the family physician, a client is diagnosed with a bunion on the lateral side of the great toe, at the metatarsophalangeal joint. Which statement should the nurse include in the teaching session?

"Some bunions are congenital; others are caused by wearing shoes that are too short or narrow."

The nurse and a nursing student are admitting a client with a malignant glioma brain tumor preoperatively for resection of the tumor. The nursing student asks the nurse, " I was told these types of tumors have a very poor prognosis. Why is the tumor being resected?" Which rationale for this intervention is true?

"Surgical resection of the tumor will decrease intracranial pressure."

A nurse and nursing student are caring for a client recovering from a lumbar puncture yesterday. The client reports a headache despite being on bedrest overnight. The physician plans an epidural blood patch this morning. The student asks how this will help the headache. The correct reply from the nurse is which of the following?

"The blood will seal the hole in the dura and prevent further loss of cerebral spinal fluid."

A client with Guillain-Barré syndrome has paralysis affecting the respiratory muscles and requires mechanical ventilation. When the client asks the nurse about the paralysis, how should the nurse respond?

"The paralysis caused by this disease is temporary."

An older adult man has been diagnosed with a femoral head fracture after falling outside his home, and his health care provider has chosen open reduction with internal fixation (ORIF). How should the nurse best explain this procedure to the patient?

"The surgeon will use pins and rods to keep your bones in place until they heal."

The nurse is providing health education to a client recently diagnosed with a brain tumor. During the appointment, the client states, "I'm really worried that I will have a seizure while I am at work or with my kids. Should I be concerned about this?" How should the nurse respond?

"There is a risk for seizures in people who have this diagnosis. What have you already discussed with your primary health care provider regarding management of seizures?"

A nurse is providing education about migraine headaches to a community group. The cause of migraines has not been clearly demonstrated, but is related to vascular disturbances. A member of the group asks about familial tendencies. The nurse's correct reply will be which of the following?

"There is a strong familial tendency."

The older client asks the nurse how best to maintain strong bones. The best response by the nurse is:

"Weight-bearing exercises can strengthen bones."

The nurse is assessing the client's mental status . Which question will the nurse include in the assessment?

"Who is the president of the United States?"

A client who has just been diagnosed with mixed muscular dystrophy asks the nurse about the usual course of this disease. How should the nurse respond?

"You may experience progressive deterioration in all voluntary muscles."

A client whose physical findings suggest a hyperpituitary condition undergoes an extensive diagnostic workup. Test results reveal a pituitary tumor, which necessitates a transsphenoidal hypophysectomy. The evening before the surgery, the nurse reviews preoperative and postoperative instructions given to the client earlier. Which postoperative instruction should the nurse emphasize?

"You must avoid coughing, sneezing, and blowing your nose."

A patient who has been diagnosed with colon cancer is scheduled to undergo positron emission tomography (PET) to search for metastases. In preparation for this diagnostic procedure, what teaching point should the nurse provide to the patient?

"You'll be given a radioactive substance that will be measured during the test."

A client with osteoporosis is prescribed calcitonin (Miacalcin) 100 units subcutaneously. The medication is available 200 units per ml. How many milliliters will the nurse administer to the client?

0.5

Hyperglycemia for a patient with a TBI may worsen the outcome of recovery. Select a serum glucose level that is considered critical.

180 mg/dL

A nurse assesses the patient's LOC using the Glasgow Coma Scale. What score indicates severe impairment of neurologic function?

3

When planning care for a client with a head injury, which position should the nurse include in the care plan to enhance client outcomes?

30-degree head elevation

A client is to undergo surgery to repair a ruptured Achilles tendon and application of a brace. The client demonstrates understanding of his activity limitations when he states that he will need to wear the brace for which length of time?

6 to 8 weeks

A client with cerebral metastasis suddenly experiences a seizure for which phenytoin 10 mg/kg intravenously is ordered as an initial loading dose. The client weighs 132 pounds. How many milligrams of phenytoin should the client receive? Enter the number ONLY.

600

Which Glasgow Coma Scale score is indicative of a severe head injury?

7

The nurse is caring for a patient diagnosed with an acute subdural hematoma following a craniotomy. The nurse is preparing to administer an IV dose of dexamethasone (Decadron). The medication is available in a 20-mL IV bag and ordered to be infused over 15 minutes. At what rate (mL/hr) will the nurse set the infusion pump?

80

A client is receiving intravenous (IV) mannitol to prevent increased intracranial pressure. The order is for mannitol 1.5 grams per kg of body weight IV now. The client weighs 143 lbs (65 kg). How many grams will the nurse administer to the client? Enter the correct number ONLY.

97.5

A nurse is assisting with a community screening for people at high risk for stroke. To which of the following clients would the nurse pay most attention?

A 60-year-old African-American man

An 82-year-old woman was diagnosed with osteoporosis several years and has lost more than 5 inches in height over the past several years as a result of structural changes in her vertebrae. The nurse should recognize that this woman is experiencing the effects of:

A compression fracture

A 37-year-old male is brought to the clinic by his wife because the patient is experiencing loss of motor function and sensation. After initial neurological assessment, the health care provider suspects the patient has a spinal cord tumor and hospitalizes him for diagnostic testing. In preparation for diagnostic studies, the nurse will inform the patient that the most commonly used study to diagnosis spinal cord compression from a tumor is what?

A magnetic resonance imaging (MRI) scan

A client in the surgical intensive care unit has skeletal tongs in place to stabilize a cervical fracture. Protocol dictates that pin care should be performed each shift. When providing pin care for the client, which finding should the nurse report to the physician?

A small amount of yellow drainage at the left pin insertion site

A patient had a carotid endarterectomy yesterday and when the nurse arrived in the room to perform an assessment, the patient states, "All of a sudden, I am having trouble moving my right side." What concern should the nurse have about this complaint?

A thrombus formation at the site of the endarterectomy

A 30-year-old primiparous woman has been admitted in early labor. The obstetrical nurse has read on the patient's prenatal record that she has a history of seizures. The nurse should understand that seizures most often occur as a result of:

Abnormal activity in the cerebral cortex

Which statement describes paresthesia?

Abnormal sensations

The nurse is admitting a patient to the unit who is diagnosed with a lower motor neuron lesion. What clinical manifestations would this patient most likely exhibit?

Absent or decreased reflexes

A 34-year-old patient is diagnosed with relapsing-remitting MS. The nurse explains to the patient's family that they should expect:

Acute attacks with full recovery or residual deficit upon recovery.

Which body movement involves moving toward the midline?

Adduction

A medical nurse has provided care for many patients who have bone disorders, and the nurse integrates health education around the maintenance of bone health into care. What should the nurse recommend to patients in an effort to promote bone health?

Adequate dietary intake of calcium

During a Tensilon test to determine if a patient has myasthenia gravis, the patient complains of cramping and becomes diaphoretic. Vital signs are BP 130/78, HR 42, and respiration 18. What intervention should the nurse prepare to do?

Administer atropine to control the side effects of edrophonium.

Two days after surgery to amputate his left lower leg, a client states that he has pain in the missing extremity. Which action by the nurse is most appropriate?

Administer medication, as ordered, for the reported discomfort.

An older adult client slipped on an area rug at home and fractured the left hip. The client is unable to have surgery immediately and is having severe pain. What interventions should the nurse provide for the patient to minimize energy loss in response to pain?

Administer prescribed analgesics around-the-clock.

A client is transferred to the intensive care unit after evacuation of a subdural hematoma. Which nursing intervention reduces the client's risk of increased intracranial pressure (ICP)?

Administering a stool softener as ordered

A client is admitted with acute osteomyelitis that developed after an open fracture of the right femur. When planning this client's care, the nurse should anticipate which measure?

Administering large doses of I.V. antibiotics as ordered

Which interventions are appropriate for a client with increased intracranial pressure (ICP)? Select all that apply.

Administering prescribed antipyretics Maintaining aseptic technique with an intraventricular catheter Frequent oral care

The nurse is providing care to a client following a knee arthroscopy. Which of the following would the nurse expect to include in the client's plan of care?

Administering the prescribed analgesic.

A college student goes to the infirmary with a fever, headache, and a stiff neck. The nurse suspects the student may have meningitis and has the student transferred to the hospital. If the diagnosis is confirmed, what should the nurse institute for those who have been in contact with this student? Select all that apply.

Administration of rifampin (Rifadin) Administration of ciprofloxacin hydrochloride (Cipro) Administration of ceftriaxone sodium (Rocephin)

Which term refers to the failure to recognize familiar objects perceived by the senses?

Agnosia

Which term refers to the inability to recognize objects through a particular sensory system?

Agnosia

The nurse is caring for a client with a hip fracture. The physician orders the client to start taking a bisphosphonate. Which medication would the nurse document as given?

Alendronate

The primary functions of cartilage are to reduce friction between articular surfaces, absorb shocks, and reduce stress on joint surfaces. Where in the human body is cartilage found?

All options are correct.

A community health nurse is conducting a home visit to a male patient who has Parkinson's disease (PD). The nurse has been working closely with the patient and his wife to prevent the many possible complications of the disease. What recommendation should the nurse make to improve nutrition?

Allot a large amount of time for each meal of the day

A client suffers a head injury. The nurse implements an assessment plan to monitor for potential subdural hematoma development. Which manifestation does the nurse anticipate seeing first?

Alteration in level of consciousness (LOC)

Which is a late sign of increased intracranial pressure (ICP)?

Altered respiratory patterns

Which disease includes loss of motor neurons in the anterior horns of the spinal cord and motor nuclei of the lower brain stem?

Amyotrophic lateral sclerosis

The school nurse notes a 6-year-old running across the playground with friends. The child stops in mid-stride, freezing for a few seconds. Then the child resumes his progress across the playground. The school nurse suspects what in this child?

An absence seizure

A nurse leader is coordinating care for a group of medical-surgical patients. What patient should the nurse recognize as being at the highest risk for the development of osteomyelitis?

An elderly patient with an infected pressure ulcer in the sacral area

A client is experiencing muscle weakness in the upper extremities. The client raises an arm above the head but then loses the ability to maintain the position. Muscular dystrophy is suspected. Which diagnostic test would evaluate muscle weakness or deterioration?

An electromyography

A patient sustained a head trauma in a diving accident and has a cerebral hemorrhage located within the brain. What type of hematoma is this classified as?

An intracerebral hematoma

Which medication classification is prescribed when allergy is a factor causing the skin disorder?

Antihistamines

A patient is recovering in the hospital following a total hip replacement that was performed 2 days ago. In an effort to prevent the common complications associated with the surgical procedure, the nurse should implement which of the following interventions, as ordered?

Application of sequential compression devices

Which nursing intervention is appropriate for a client with double vision in the right eye due to MS?

Apply an eye patch to the right eye.

Which of the following nursing actions is most important in caring for the client following an arthrogram?

Apply ice to the joint.

A patient with spinal cord injury has a nursing diagnosis of altered mobility. Which of the following would be included as an appropriate nursing intervention to prevent deep vein thrombosis (DVT) from occurring?

Applying thigh-high elastic stockings

A patient is scheduled for a procedure that will allow the physician to visualize the knee joint in order to diagnose the patient's pain. What procedure will the nurse prepare the patient for?

Arthroscopy

The nurse is preparing a client for a surgical procedure that will allow visualization of the extent of joint damage of the knee for a client with rheumatoid arthritis and also obtain a sample of synovial fluid. What procedure will the nurse prepare the client for?

Arthroscopy

A nurse is caring for a client who has returned to his room after a carotid endarterectomy. Which action should the nurse take first?

Ask the client if he has trouble breathing.

The nurse would include which of the following in a neurological assessment?

Ask the client to plantar flex the toes.

A client is ordered to undergo CT of the brain with IV contrast. Before the test, the nurse should complete which action first?

Assess the client for medication allergies.

Which nursing intervention is the priority for a client in myasthenic crisis?

Assessing respiratory effort

A high school student who was injured in a football game presents with knee pain with internal rotation of the foot. Which of the following is an inappropriate nursing intervention?

Assist the client to "walk" off the pain.

Which interventions would be recommended for a client with dysphagia? Select all that apply.

Assist the client with meals. Test the gag reflex before offering food or fluids. Allow ample time to eat.

A nurse is caring for a client who has sustained ligament and a meniscal injury to the knee. Which action would be most appropriate to allow the client to progress without causing further injury?

Assist with a gradual introduction of activity.

A patient is diagnosed with an intracerebral tumor. The nurse knows that the diagnosis may include which of the following? Select all that apply.

Astrocytoma Ependymoma Medulloblastoma

Which term refers to the inability to coordinate muscle movements, resulting difficulty walking?

Ataxia

The nurse who is taking care of a patient with lower motor neuron destruction documents on the electronic health record that the muscle tone in the lower legs is:

Atonic

Which drug should be available to counteract the effect of edrophonium chloride?

Atropine

Lesions in the temporal lobe may result in which type of agnosia?

Auditory

Which phase of a migraine headache usually lasts less than an hour?

Aura

You are a neurotrauma nurse working in a neuro ICU. What would you know is an acute emergency and is seen in clients with a cervical or high thoracic spinal cord injury after the spinal shock subsides?

Autonomic dysreflexia

A client who has sustained a fracture reports an increase in pain and decreased function of the affected extremity. The nurse would suspect which of the following?

Avascular necrosis

A nurse is providing education to a client with newly diagnosed multiple sclerosis (MS). Which of the following will the nurse include?

Avoid hot temperatures.

The nurse is caring for a 35-year-old man diagnosed with a back strain. What would be a priority point of discussion with this patient?

Avoid lifting heavy weights without assistance.

The nurse is caring for a client who was diagnosed with a glioma 5 months ago. Today, the client was brought to the emergency department by his caregiver because he collapsed at home. The nurse suspects late signs of rising intracranial pressure (ICP) when which blood pressure and pulse readings are noted?

BP = 175/45 mm Hg; HR = 42 bpm

A patient is admitted to the emergency room with a fractured skull sustained in a motorcycle accident. The nurse notes fluid leaking from the patient's ears. The nurse knows this is a probable sign of which type of skull fracture?

Basilar

Extensive diagnostic testing has resulted in a patient's diagnosis of a benign brain tumor. When providing care for this patient, the nurse should be cognizant of which of the following characteristics of benign brain tumors?

Benign brain tumors can slowly grow into an area of vital brain function.

A patient has stepped in a hole in the yard, causing an ankle injury. The ankle is edematous and painful to palpation. How long should the nurse inform the patient that the acute inflammatory stage will last?

Between 24 and 48 hours

Which of the following is the first-line medication that would be used to treat and prevent osteoporosis?

Bisphosphonates

The nurse is discussing an older adult's risk for skeletal fractures with a group of students assigned to the clinical area. Which of the following would the nurse most likely explain as the underlying reason for the increased risk?

Bone resorption is more rapid than bone formation.

A patient who has suffered a stroke is unable to maintain respiration and is intubated and placed on mechanical ventilator support. What portion of the brain is most likely responsible for the inability to breathe?

Brain stem

An older client complains of a constant headache. A physical examination shows papilledema. What may the symptoms indicate in this client?

Brain tumor

The nurse is caring for a patient in the emergency department with a diagnosed epidural hematoma. What procedure will the nurse prepare the patient for?

Burr holes

When assessing a client who has experienced a spinal injury, the nurse notes diaphragmatic breathing and loss of upper limb use and sensation. At what level does the nurse anticipate the injury has occurred?

C5

Which hormone inhibits bone reabsorption and increases calcium deposit in the bone?

Calcitonin

Which medication directly inhibits osteoclasts, thereby reducing bone loss and increasing bone mass density (BMD)?

Calcitonin

Which of the following inhibits bone resorption and promotes bone formation?

Calcitonin

A nurse is caring for a client diagnosed with a cerebral aneurysm who reports a severe headache. Which action should the nurse perform?

Call the physician immediately.

An important nursing assessment, post fracture, is to evaluate neurovascular status. Therefore, the nurse should check for:

Capillary refill

When performing a physical assessment of a client, the client reports numbness, tingling, and pain when the nurse percusses lightly over the median nerve. The nurse recognizes that this finding is consistent with:

Carpal tunnel syndrome

A patient recently noted difficulty maintaining his balance and controlling fine movements. The nurse explains that the provider will order diagnostic studies for the part of his brain known as the:

Cerebellum

The nurse is caring for a client who has been hospitalized for investigation of a sudden change in gait due to loss of balance and coordination. A magnetic resonance imaging scan reveals the client has a brain tumor. On or close to which brain structure is the tumor most likely situated?

Cerebellum

What part of the brain controls and coordinates muscle movement?

Cerebellum

The physician's office nurse is caring for a client who has a history of a cerebral aneurysm. Which diagnostic test does the nurse anticipate to monitor the status of the aneurysm?

Cerebral angiography

Damage to the brain from traumatic injury can be divided into primary and secondary injuries. Which of the following is cause of a secondary injury associated with brain injury? Select all that apply.

Cerebral edema Ischemia Infection Seizures Hyperthermia

A nurse is preparing a client for a lumbar puncture. The client has heard about post-lumbar puncture headaches and asks what causes them. The nurse tells the client that these headaches are caused by which of the following?

Cerebral spinal fluid leakage at the puncture site

The nurse is assisting the physician in completing a lumbar puncture. Which would the nurse note as a concern?

Cerebrospinal fluid is cloudy in nature.

The nurse is caring for a client who underwent surgery to remove a spinal cord tumor. When conducting the postoperative assessment, the nurse notes the presence of a bulge at the surgical site. The nurse suspects the client is experiencing what complication from the surgery?

Cerebrospinal fluid leakage

When an infection is bloodborne, the manifestations include which symptom?

Chills

Which of the following outcomes would be most appropriate to include in the plan of care for a client diagnosed with a muscular dystrophy?

Client participates in activities of daily living using adaptive devices.

The nurse identifies a nursing diagnosis of self-care deficit, bathing related to motor impairment and decreased cognitive function for a client with cerebral metastasis. Which outcome would the nurse most likely identify on this client's plan of care?

Client participates in daily hygiene activities with assistive devices.

A client with a brain tumor is experiencing changes in cognition that require the nurse to reorient the client frequently. When performing this task, which devices would be appropriate for the nurse to use? Select all that apply.

Client's clothing Picture of the client's family Clock Calendar

A nurse is assessing reflexes in a patient with hyperactive reflexes. When the patient's foot is abruptly dorsiflexed, it continues to "beat" two to three times before setting into a resting position. How would the nurse document this finding?

Clonus

What is the term for a rhythmic contraction of a muscle?

Clonus

Which is a neurovascular problem caused by pressure within a muscle area that increases to such an extent that microcirculation diminishes?

Compartment syndrome

A client is admitted to an acute care facility after an episode of status epilepticus. After the client is stabilized, which factor is most beneficial in determining the potential cause of the episode?

Compliance with the prescribed medication regimen

A patient with generalized seizure disorder has just had a seizure. The nurse would assess for what characteristic associated with the postictal state?

Confusion

A community health nurse is conducting a workshop for unlicensed care providers who work in a chain of long-term care facilities. The nurse is teaching the participants about the signs and symptoms of stroke. What signs and symptoms should the nurse identify? Select all that apply.

Confusion Sudden numbness Visual disturbances

Which is the primary medical management of arthropod-borne virus (arboviral) encephalitis?

Controlling seizures and increased intracranial pressure

A client comes to the Emergency Department complaining of localized pain and swelling of his lower leg. Ecchymotic areas are noted. History reveals that the client got hit in the leg with a baseball bat. Which of the following would the nurse suspect as most likely?

Contusion

A client comes to the emergency department complaining of localized pain and swelling of his lower leg. Ecchymotic areas are noted. History reveals that the client got hit in the leg with a baseball bat. Which of the following would the nurse suspect as most likely?

Contusion

A client presents to the emergency department with an open fracture. What is the first action the nurse should take?

Cover the exposed bone with sterile dressing.

The patient presents to the emergency room with an open fracture of the femur. Which action would the nurse implement to prevent the most serious complication of an open fracture?

Cover the wound with a sterile dressing to prevent infection

Which condition is a rare, transmissible, progressive fatal disease of the central nervous system characterized by spongiform degeneration of the gray matter of the brain?

Creutzfeldt-Jakob disease

A female patient who is recovering from a stroke has begun eating a minced and pureed diet after passing the speech pathologist's swallowing assessment. This morning, the nurse set up the patient with her breakfast tray and later noticed that the woman was swallowing her food well but dribbling small amounts of food out of affected side of her mouth. How should the nurse follow up this observation?

Cue the patient to the fact that she is dribbling food while commending her for eating.

When the nurse observes that the patient has extension and external rotation of the arms and wrists, and extension, plantar flexion, and internal rotation of the feet, she records the patient's posturing as which of the following?

Decerebrate

A nurse observes an abnormal posture response in an unconscious patient. She documents "extension and outward rotation of the upper extremities and plantar flexion of the feet." She is aware that this posture is a clinical indicator of which of the following?

Decerebrate positioning implying severe dysfunction and brain pathology

A nurse is working in the neurologic intensive care unit and admits from the emergency department a patient with a severe head injury. Upon entering the room, the nurse observes that the patient is positioned like part A of the accompanying image. Which posturing is the patient exhibiting?

Decorticate

A nurse who provides care in an extended-care setting works primarily with adults aged 75 and older. When planning care and implementing safety measures for this population, the nurse should be cognizant of which of the following normal, age-related changes to the musculoskeletal system?

Decreased bone mass

A patient has a deficiency of the neurotransmitter serotonin. The nurse is aware that this deficiency can lead to:

Depression

Low levels of the neurotransmitter serotonin lead to which of the following disease processes?

Depression

Which term refers to the shaft of the long bone?

Diaphysis

Which of the following would lead a nurse to suspect that a client has a rotator cuff tear?

Difficulty lying on affected side

The nurse is caring for a client with a history of transient ischemic attacks (TIAs) and moderate carotid stenosis who has undergone a carotid endarterectomy. Which postoperative finding would cause the nurse the most concern?

Difficulty swallowing

Which is a sympathetic effect of the nervous system?

Dilated pupils

The nurse is assessing a client newly diagnosed with myasthenia gravis. Which of the following signs would the nurse most likely observe?

Diplopia and ptosis

Which of the following disorders results in widespread hemorrhage and microthrombosis with ischemia?

Disseminated intravascular coagulation (DIC)

The nurses instructs the client not to cross their legs and to have someone assist with tying their shoes. Which additional instruction should the nurse provide to client?

Do not flex the hip more than 90 degrees

Impaired balance and uncontrolled tremors of Parkinson's disease is correlated with which neurotransmitter?

Dopamine

The nurse is caring for a client hospitalized after a motor vehicle accident. The client has a comorbidity of Parkinson's disease. Why should the nurse closely monitor the condition and the drug regimen of a client with Parkinson's disease?

Drugs administered may cause a wide variety of adverse effects.

What term refers to a flexion deformity caused by a slowly progressive contracture of the palmar fascia?

Dupuytren contracture

Which term refers to a flexion deformity caused by a slowly progressive contracture of the palmar fascia?

Dupuytren contracture

A client with a brain tumor is complaining of a headache upon awakening. Which nursing action would the nurse take first?

Elevate the head of the bed.

A client undergoes a craniotomy with supratentorial surgery to remove a brain tumor. On the first postoperative day, the nurse notes the absence of a bone flap at the operative site. How should the nurse position the client's head?

Elevated 30 degrees

A patient admitted for the treatment of a nondepressed skull fracture has been leaking clear fluid from his nose, and glucose testing confirms that it is cerebrospinal fluid (CSF). This development necessitates what nursing action?

Elevating the head of the bed to 30 degrees

A client with arterial insufficiency undergoes below-knee amputation of the right leg. Which action should the nurse include in the postoperative care plan?

Elevating the stump for the first 24 hours

Which general nursing measure is used for a client with a fracture reduction?

Encourage participation in ADLs

A client has undergone a lumbar puncture as part of a neurological assessment. The client is put under the care of a nurse after the procedure. Which important postprocedure nursing intervention should be performed to ensure the client's maximum comfort?

Encourage the client to drink liberal amounts of fluids

The nurse is expecting to admit a client with a diagnosis of meningitis. While preparing the client's room, which of the following would the nurse most likely have available?

Equipment to maintain infection control precautions

The nurse is caring for a client with aphasia. Which strategy will the nurse use to facilitate communication with the client?

Establishing eye contact

A client with a traumatic brain injury has developed increased intracranial pressure resulting in dibetes insipidus. While assessing the client, the nurse expects which of the following findings?

Excessive urine output and decreased urine osmolality

The nurse is caring for a client with an inoperable brain tumor. What teaching is important for the nurse to do with these clients?

Explaining hospice care and services

The nurse is providing a health promotion session on the risk factors for various health conditions. When discussing brain tumors, the nurse should include in the session which known risk factors for brain tumors? Select all that apply.

Exposure to certain chemicals Having ionizing radiation

A nurse is reviewing a client's medical record and finds that the client has a spinal cord tumor that involves the vertebral bodies. The nurse identifies this as which type of spinal tumor?

Extradural

Level of consciousness (LOC) can be assessed based on criteria in the Glasgow Coma Scale (GCS). Which of the following indicators are assessed in the GCS? Select all that apply.

Eye opening Verbal response Motor response

A nurse is communicating with a client who has aphasia after having a stroke. Which action should the nurse take?

Face the client and establish eye contact.

Bell palsy is a disorder of which cranial nerve?

Facial (VII)

A patient sustained an open fracture of the femur 24 hours ago. While assessing the patient, the nurse observes the patient is having difficulty breathing, and oxygen saturation decreases to 88% from a previous 99%. What does the nurse understand is likely occurring with this patient?

Fat emboli

The client with a fractured left humerus reports dyspnea and chest pain. Pulse oximetry is 88%. Temperature is 100.2 degrees Fahrenheit (38.5 degrees Centigrade); heart rate is 110 beats per minute; respiratory rate is 32 breaths per minute. The nurse suspects the client is experiencing:

Fat embolism syndrome

A nurse is caring for a client with deteriorating neurologic status. The nurse is performing an assessment at the beginning of the shift that reveals a falling blood pressure and heart rate, and the client makes no motor response to stimuli. Which documentation of neuromuscular status is most appropriate?

Flaccidity

A client has been diagnosed as having global aphasia. The nurse recognizes that the client will be unable to do perform which action?

Form understandable words and comprehend spoken words

A client has been diagnosed as having global aphasia. The nurse recognizes that the client will be unable to perform which action?

Form words that are understandable or comprehend spoken words

From which direction should a nurse approach a client who is blind in the right eye?

From the left side of the client

A stroke victim is experiencing memory loss and impaired learning capacity. The nurse knows that brain damage has most likely occurred in which lobe?

Frontal

Which cerebral lobes is the largest and controls abstract thought?

Frontal

Which lobe of the brain is responsible for concentration and abstract thought?

Frontal

A patient sustained a head injury during a fall and has changes in personality and affect. What part of the brain does the nurse recognize has been affected in this injury?

Frontal lobe

Which is a nonmodifiable risk factor for ischemic stroke?

Gender

A 42-year-old man presents at the clinic complaining of pain in his heel so bad it inhibits his ability to walk. The patient is diagnosed with plantar fasciitis. Patient teaching provided by the nurse would include what instructions to decrease the pain associated with this condition?

Gently stretching the foot and the Achilles tendon

Which term refers to a fracture in which one side of a bone is broken and the other side is bent?

Greenstick

A patient is having a lumbar puncture and the physician has removed 20 mL of cerebrospinal fluid. What nursing intervention is a priority after the procedure?

Have the patient lie flat for 6 hours.

A patient has recently been diagnosed with an acoustic neuroma. The nurse helps the patient understand that:

Hearing loss usually occurs.

A client is experiencing dysphagia following a stroke. Which measure may be taken by the nurse to ensure that the client's diet allows for easy swallowing?

Help the client sit upright when eating and feed slowly.

A health care provider asks a nurse to assess a patient being evaluated for aseptic meningitis for a positive Brudzinski sign. Which of the following actions should the nurse take?

Help the patient flex his neck and observe for flexion of the hips and knees.

An emergency department (ED) nurse has administered an ordered bolus of tissue plasminogen activator (tPA) to a male patient who was diagnosed with stroke. During the administration of tPA, the nurse should prioritize assessments related to what problem?

Hemorrhage

A client is diagnosed with a brain angioma. When teaching the client about the risks associated with this type of brain tumor, the nurse would educate about signs and symptoms associated with which condition?

Hemorrhagic stroke

A physician orders several drugs for a client with hemorrhagic stroke. Which drug order should the nurse question?

Heparin sodium

A nurse practitioner who works with many older adult patients integrates screening for osteoporosis into health promotion activities. What screening tool is most clinically useful when screening for osteoporosis?

Hip bone mineral density (BMD) testing

The provider asks a nurse to test a patient for Tinel's sign to diagnose carpal tunnel syndrome. The nurse asked the patient to:

Hold his palm up while the nurse percussed over the median nerve.

Which disturbance results in loss of half of the visual field?

Homonymous hemianopsia

Which term will the nurse use when referring to blindness in the right or left half of the visual field in both eyes?

Homonymous hemianopsia

The nursing instructor gives their students an assignment of making a plan of care for a client with Huntington's disease. What would be important for the students to include in the teaching portion of the care plan?

How to facilitate tasks such as using both hands to hold a drinking glass

A patient with a primary diagnosis of colorectal cancer has developed bone metastases in recent weeks. When reviewing this patient's daily blood work, the nurse should be aware that secondary bone tumors create a significant risk of what electrolyte imbalance?

Hypercalcemia

A nurse is providing education to a community group about ischemic strokes. One group member asks if there are ways to reduce the risk for stroke. Which of the following is a risk factor that can be modified?

Hypertension

A classic indicator of edema and alveolar hemorrhage associated with FES is:

Hyperventilation

Which of the following areas of the brain are responsible for temperature regulation?

Hypothalamus

The nurse is caring for a patient on the neurological unit who is in status epilepticus. What medication does the nurse anticipate being given to halt the seizure?

IV diazepam

Which type of fracture occurs when a bone fragment is driven into another bone fragment?

Impacted

A 55-year-old male patient has been admitted to the hospital with a gastrointestinal bleed, and the patient has just experienced a generalized seizure that may be attributable to alcohol withdrawal. When providing immediate care during the patient's seizure, what nursing diagnosis should be prioritized?

Impaired gas exchange

The nurse is providing discharge teaching to a client with a spinal cord tumor and instructs the client to avoid hot water bottles and heating blankets for what reason?

Impaired sensory perception

A patient has just returned to the unit from the PACU after surgery for a tumor within the spine. The patient complains of pain. When positioning the patient for comfort and to reduce injury to the surgical site, the nurse will position the patient in what position?

In a flat side-lying position

Which type of fracture involves a break through only part of the cross-section of the bone?

Incomplete

The nurse is caring for a patient postoperatively after intracranial surgery for the treatment of a subdural hematoma. The nurse observes an increase in the patient's blood pressure from the baseline and a decrease in the heart rate from 86 to 54. The patient has crackles in the bases of the lungs. What does the nurse suspect is occurring?

Increased ICP

Which assessment finding would cause the nurse to suspect compartment syndrome in the client following a bone biopsy?

Increased diameter of the calf

A fall during a rock climbing expedition this morning has caused a 28-year-old woman to develop an epidural hematoma. Immediate treatment is being organized by the emergency department team because this woman faces a risk of serious neurological damage as a result of:

Increased intracranial pressure (ICP)

A patient has been admitted to the intensive care unit (ICU) for the treatment of bacterial meningitis. The ICU nurse is aware of the need for aggressive treatment and vigilant nursing care because meningitis has the potential to cause what sequela?

Increased intracranial pressure (ICP)

The nurse is seeing a female client who has been diagnosed with a pituitary adenoma. During the clinic visit, the client tells the nurse that she has been having irregular menstrual periods despite having very regular menstrual periods all her life. The nurse knows this physiological change is likely related to which characteristic of this type of brain tumor?

Increased prolactin levels

The nurse is caring for a client with Guillain-Barré syndrome. Which assessment finding would indicate the need for oral suctioning?

Increased pulse rate, adventitious breath sounds

A client is receiving an IV infusion of mannitol (Osmitrol) after undergoing intracranial surgery to remove a brain tumor. To determine whether this drug is producing its therapeutic effect, the nurse should consider which finding most significant?

Increased urine output

The nurse is caring for a patient who sustained an open fracture of the right femur in an automobile accident. What does the nurse understand is the most serious complication of an open fracture?

Infection

A child has fallen off a swing on a playground and sustained a greenstick (partial) fracture of his radius. Because the child is otherwise healthy, healing has begun promptly after his injury. Which of the following is the first stage of bone healing?

Inflammation and hematoma formation

Nurses are performing hourly neurological assessments of a patient who is postoperative day 1 following spinal surgery that was performed to treat spinal cord compression (SCC). The nurse's most recent assessment shows a marked reduction in motor and sensory function. How should the nurse best follow-up this assessment finding?

Inform the care provider promptly about this sudden change.

A female client is at risk for developing osteoporosis. Which action will reduce the client's risk?

Initiating weight-bearing exercise routines

The nurse is caring for a client with a head injury. The client is experiencing CSF rhinorrhea. Which order should the nurse question?

Insertion of a nasogastric (NG) tube

What is the only known risk factor for brain tumors?

Ionizing radiation

A client with a brain tumor experiences projectile vomiting. The nurse integrates understanding of this occurrence as resulting from which of the following?

Irritation of the medullary vagal centers

The nurse is caring for a client admitted with a stroke. Imaging studies indicate an embolus partially obstructing the right carotid artery. What type of stroke does the nurse know this client has?

Ischemic

The nurse working on a neurological unit is mentoring a nursing student. The student asks about a client who has sustained a primary and and secondary brain injury. The nurse correctly tells the student which of the following, related to the primary injury?

It results from initial damage to the brain from the traumatic event

A bone graft may be used for which of the following reasons? Select all that apply.

Joint stabilization Defect filling Stimulation of bone healing

A male client who has undergone a cervical discectomy is being discharged with a cervical collar. Which of the following would be most appropriate to include the client's discharge plan?

Keeping the head in a neutral position

Which of the following is an example of a hinge joint?

Knee

The nurse is providing postoperative care for a client who just underwent surgery to remove a metastatic intramedullary tumor. On postoperative day 3, the client states, "I am really looking forward to going running again, it had become too difficult because of the loss of feeling in my feet." Which should the nurse address in the client's care plan?

Knowledge deficit

The nurse is performing an assessment on an older adult patient and observes the patient has an increased forward curvature of the thoracic spine. What does the nurse understand this common finding is known as?

Kyphosis

A patient visits an orthopedic specialist because of pain that he feels beginning in his low back and radiating behind his right thigh and down below his right knee. The doctor suspects a diagnosis of sciatica. The nurse knows that the origin of the pain is between these intervertebral disks:

L4, L5, and S1

A nurse is assisting during a lumbar puncture. How should the nurse position the client for this procedure?

Lateral recumbent, with chin resting on flexed knees

What does the nurse recognize as the earliest sign of serious impairment of brain circulation related to increasing ICP?

Lethargy and stupor

A client has experienced an ischemic stroke that has damaged the frontal lobe of his brain. Which of the following deficits does the nurse expect to observe during assessment?

Limited attention span and forgetfulness

The nurse practitioner prescribes the medication of choice for an MS patient who is experiencing disabling episodes of muscles spasms, especially at night. Which of the following is the drug most likely prescribed in this scenario?

Lioresal

Which factor inhibits fracture healing?

Local malignancy

A client with a concussion is discharged after the assessment. Which instruction should the nurse give the client's family?

Look for signs of increased intracranial pressure

Which of the following is an age-related change to the musculoskeletal system?

Loss of bone mass

Bone density testing will be completed for the client with post-polio syndrome. The nurse teaches the client bone density testing is used to identify what potential complication?

Low bone mass and osteoporosis

Which area of the spinal column is subject to the greatest mechanical stress and degenerative changes?

Lower lumbar

A nurse is reading a journal article about spinal cord tumors and metastasis from other primary sites. The nurse demonstrates understanding of the article when identifying which primary sites as commonly metastasizing to the spinal cord? Select all that apply.

Lung Breast Gastrointestinal tract

An 83-year-old woman suffers a stroke at home and is hospitalized for treatment and management. Which of the following diagnostic procedures would be best to visualize the extent of damage?

Magnetic resonance angiography (MRA)

A client with spinal cord compression from a tumor must undergo diagnostic testing. Which of the following is the most likely procedure for this client?

Magnetic resonance imaging

A 13-year-old patient is admitted to the pediatric unit with a suspected brain tumor. The nurse should understand that which diagnostic test is the most helpful in the diagnosis of brain tumors?

Magnetic resonance imaging (MRI)

Which diagnostic is most commonly used for spinal cord compression?

Magnetic resonance imaging (MRI)

The nurse in the neurologic ICU is caring for a client who sustained a severe brain injury. Which nursing measures will the nurse implement to help control intracranial pressure (ICP)?

Maintain cerebral perfusion pressure from 50 to 70 mm Hg

An older adult patient has been brought to the emergency department (ED) after being found unconscious by a neighbor. What action should be the ED nurse's highest priority in the care of this patient?

Maintain the patency of the patient's airway.

The nurse is caring for a client diagnosed with a hemorrhagic stroke. The nurse recognizes that which intervention is most important?

Maintaining a patent airway

The nurse is planning the care of a patient with a TBI in the neurosurgical ICU. In developing the plan of care, what interventions should be a priority? Select all that apply.

Making nursing assessments Setting priorities for nursing interventions Anticipating needs and complications Initiating rehabilitation

A patient is brought to the trauma center by ambulance after sustaining a high cervical spinal cord injury 1½ hours ago. What medication does the nurse know will be given to prevent further spinal cord damage?

Methylprednisolone (Solu-Medrol)

A patient has sustained a long bone fracture. The nurse is preparing a care plan for this patient. Which nursing action should the nurse include in the care plan to enhance fracture healing?

Monitor color, temperature, and pulses of the affected extremity.

A nurse is reviewing a CT scan of the brain, which states that the client has arterial bleeding with blood accumulation above the dura. Which of the following facts of the disease progression is essential to guide the nursing management of client care?

Monitoring is needed as rapid neurologic deterioration may occur.

A nurse is continually monitoring a client with a traumatic brain injury for signs of increasing intracranial pressure. The cranial vault contains brain tissue, blood, and cerebrospinal fluid; an increase in any of the components causes a change in the volume of the others. This hypothesis is called which of the following?

Monro-Kellie

Which of the following is considered a central nervous system (CNS) disorder?

Multiple sclerosis

Which clinical manifestation would be exhibited by a client following a hemorrhagic stroke of the right hemisphere?

Neglect of the left side

After a person experiences a closure of the epiphyses, which statement is true?

No further increase in bone length occurs.

Which of the following is the initial diagnostic in suspected stroke?

Noncontrast computed tomography (CT)

Which term refers to the failure of fragments of a fractured bone to heal together?

Nonunion

A patient 3 days postoperative from a craniotomy informs the nurse, "I feel something trickling down the back of my throat and I taste something salty." What priority intervention does the nurse initiate?

Notify the physician of a possible cerebrospinal fluid leak.

Chapter 47

Nursing Management: Patients With Cerebrovascular Disorders

Chapter 45

Nursing Management: Patients With Neurologic Trauma

An osteocalcin (bone GLA protein) level has been ordered. The nurse prepares to:

Obtain a blood specimen.

A client is diagnosed with a brain tumor. The nurse's assessment reveals that the client has difficulty interpreting visual stimuli. Based on these findings, the nurse suspects injury to which lobe of the brain?

Occipital

A nurse is completing a neurological assessment and determines that the client has significant visual deficits. A brain tumor is considered. Considering the functions of the lobes of the brain, which area will most likely contain the neurologic deficit?

Occipital

A patient is brought to the emergency room following a motor vehicle accident in which she sustained a head trauma. The patient is complaining of blindness in her left eye. The nurse would be correct in suspecting that this sensory deficit is related to damage in what cerebral lobe?

Occipital

A 77-year-old female patient who is recovering in the hospital from a total knee replacement has rung her call bell and told the nurse that she needs pain medication. When assessing the patient's pain, what principle should the nurse bear in mind?

Older adults tend to have a blunted pain sensation, so complaints should be followed-up promptly.

A patient has presented to the emergency room with a fracture of the nose that has resulted in a skin tear and involvement of the mucous membranes of the nasal passages. The emergency room nurse is aware that this description likely indicates which type of fracture?

Open

A nurse is caring for a client with an undiagnosed bone disease. When instructing on the normal process to maintain bone tissue, which process transforms osteoblasts into mature bone cells?

Ossification and calcification

An instructor is describing the process of bone development. Which of the following would the instructor describe as being responsible for the process of ossification?

Osteoblasts

The client presents to the emergency department with fever, chills, restlessness, and limited movement of a fractured jaw. The nurse interprets these findings as indicating which of the following complications?

Osteomyelitis

A 49-year-old man with a history of poorly controlled type 1 diabetes has developed osteomyelitis adjacent to a chronic diabetic ulcer on his great toe. The patient has been informed that medical treatment for osteomyelitis requires a longer course of antibiotics than most other infections because:

Osteomyelitis involves the active infection of bone tissue, which is largely avascular.

The pre-nursing class is learning about the nervous system in their anatomy class. What part of the nervous system would the students learn is responsible for digesting food and eliminating body waste?

Parasympathetic

A client presents to the emergency department stating numbness and tingling occurring down the left leg into the left foot. When documenting the experience, which medical terminology would the nurse be most correct to report?

Paresthesia

A patient comes to the clinic and informs the nurse of numbness, tingling, and a burning sensation in the arm from the elbow down to the fingers. What type of symptom would this be documented as?

Paresthesia

A nurse is taking a newly admitted patient's health history, and the patient states that she has had ongoing problems with a "pinched nerve." In addition to the potential for pain, the nurse should consider the fact that physical pressure that is placed on nerves can often lead to what?

Paresthesias

The nurse is caring for a client following foot surgery. Which nursing intervention is most important for the nurse to include in the nursing care plan?

Perform neuromuscular assessment every hour.

After striking his head on a tree while falling from a ladder, a client is admitted to the emergency department. He's unconscious and his pupils are nonreactive. Which intervention should the nurse question?

Performing a lumbar puncture

Which nerve is being assessed when the nurses asks the client to dorsiflex the ankle and extend the toes?

Peroneal

Which of the following would the nurse use to determine that a client is exhibiting signs and symptoms of chronic osteomyelitis?

Persistent draining sinus

Red bone marrow produces which of the following? Select all that apply.

Platelets White blood cells (WBCs) Red blood cells (RBCs)

The nurse recognizes that goal of treatment for metastatic bone cancer is to:

Promote pain relief and quality of life

The nurse cares for a client with Huntington disease. What intervention is a priority for safe care?

Protecting the client from falls

A client with a traumatic amputation of the right lower leg is refusing to look at the leg. Which action by the nurse is most appropriate?

Provide feedback on the client's strengths and available resources.

A 36-year-old male patient is preparing for discharge from the hospital to a rehabilitative facility 4 weeks after he suffered a spinal cord injury (SCI) during a workplace accident. The hospital nurse should be aware that the primary focus of this coming phase of the patient's recovery will be:

Providing him with the skills to perform as many activities of daily living (ADLs) as possible

The nurse is caring for a client following an aneurysm coiling procedure. The nurse documents that the client is experiencing Korsakoff syndrome. Which set of symptoms characterizes Korsakoff syndrome?

Psychosis, disorientation, delirium, insomnia, and hallucinations

The nurse is caring for a client who experienced a crushing injury of the lower extremities. Which of the following symptoms is essential to be reported to the physician?

Pulselessness

The nurse is educating the patient with low back pain about the proper way to lift objects. What muscle should the nurse encourage the patient to maximize?

Quadriceps

A client comes to the Emergency Department complaining of pain in his left ankle. The client states, "I missed a step coming down the stairs, and landed funny." The ankle is swollen and tender to the touch. Which of the following would the nurse do to help control the swelling?

Raise the left leg above the level of the heart.

The nurse is caring for a patient with Huntington's disease in the long-term care facility. What does the nurse recognize as the most prominent symptom of the disease that the patient exhibits?

Rapid, jerky, involuntary movements

A male patient with cerebrovascular accident (CVA) is prescribed medication to treat the disorder. The patient wants to know what other measures may help reduce CVA. Which of the following is an accurate suggestion for the patient?

Reduce hypertension and high blood cholesterol levels.

A nursing educator is talking with nurses about the effects of the aging process and neurologic changes. What would the educator identify as a normal neurological change that accompanies the aging process?

Reduction in cerebral blood flow (CBF)

After a fracture, during which stage or phase of bone healing is devitalized tissue removed and new bone reorganized into its former structural arrangement?

Remodeling

Which of the following is the final stage of fracture repair?

Remodeling

A client with herpes simplex virus (HSV) encephalitis is receiving acyclovir. To ensure early intervention, the nurse monitors laboratory values and urine output for which type of adverse reactions?

Renal

A community health nurse is performing a home visit to a patient with amyotrophic lateral sclerosis (ALS). The nurse should prioritize assessments related to which of the following?

Respiratory function

A client with neurologic infection develops cerebral edema from syndrome of inappropriate antidiuretic hormone (SIADH). Which is an important nursing action for this client?

Restricting fluid intake and hydration

A client with neurological infection develops cerebral edema from syndrome of inappropriate antidiuretic hormone (SIADH). Which is an important nursing action for this client?

Restricting fluid intake and hydration

The nurse practitioner is able to correlate a patient's neurologic deficits with the location in the brain affected by ischemia or hemorrhage. For a patient with a left hemispheric stroke, the nurse would expect to see:

Right-sided paralysis.

An orthopedic trauma patient is receiving lower-body skeletal traction and will continue to do so for several weeks. When planning the patient's care, what nursing diagnosis should be prioritized?

Risk for impaired skin integrity

Which of the following is an appropriate priority nursing diagnosis for the client following an arthrocentesis?

Risk for infection

A client who is disoriented and restless after sustaining a concussion during a car accident is admitted to the hospital. Which nursing diagnosis takes the highest priority in this client's care plan?

Risk for injury

Which of the following is the most important nursing diagnosis for an elderly patient diagnosed with osteoporosis?

Risk for injury related to fractures due to osteoporosis

A patient presents at the clinic complaining of back pain that goes all the way down the back of her leg to her foot. What is this type of pain referred to as?

Sciatica

Which of the following deformities causes a lateral curving deviation of the spine?

Scoliosis

A 19-year-old patient presents to the emergency room with an injury to her left ankle that occurred during a high school basketball game. She complains of limited motion and pain on walking, which increased over the last 2 hours. The nurse knows that her diagnosis is most likely which of the following?

Second-degree sprain

Following a generalized seizure in a client, which nursing assessment is a priority for detailing the event?

Seizure was 1 minute in duration including tonic-clonic activity.

The nurse is caring for a patient having a hemorrhagic stroke. What position in the bed will the nurse maintain this patient?

Semi-Fowler's

A patient has a fracture that is being treated with open rigid compression plate fixation devices. How will the progress of bone healing be monitored?

Serial x-rays

Which neurotransmitter demonstrates inhibitory action, helps control mood and sleep, and inhibits pain pathways?

Serotonin

Which laboratory study indicates the rate of bone turnover?

Serum osteocalcin

A nurse completes the Glasgow Coma Scale on a patient with traumatic brain injury (TBI). Her assessment results in a score of 6, which is interpreted as:

Severe TBI

A patient is exhibiting classic signs of a hemorrhagic stroke. What complaint from the patient would be an indicator of this type of stroke?

Severe headache

An emergency department nurse is interviewing a client with signs of an ischemic stroke that began 2 hours ago. The client reports that she had a cholecystectomy 6 weeks ago and is taking digoxin, coumadin, and labetelol. This client is not eligible for thrombolytic therapy for which of the following reasons?

She is taking coumadin.

A client is receiving hypothermic treatment for uncontrolled fever related to increased intracranial pressure (ICP). Which assessment finding requires immediate intervention?

Shivering

The nurse is providing a class on osteoporosis at the local senior citizens center. Which of the following statements related to osteoporosis should the nurse include in this presentation?

Slow discontinuation of corticosteroid therapy will halt the progression of the osteoporosis but not restore lost bone mass.

Which is a modifiable risk factor for transient ischemic attacks and ischemic strokes?

Smoking

Which is a strategy for lowering risk for osteoporosis?

Smoking cessation

A 15-year-old healthy boy presents to the emergency room with a fractured left ankle that was injured during a soccer game. The patient describes his pain as a 7 on a scale of 1 to 10. The nurse prepares the patient for the application of a cast and explains that the pain and swelling should only last several days. This information is based on an understanding of the inflammatory stage of bone healing whereby:

Soon after a fracture, fibrin will become the framework for new bone growth

The nurse is caring for a client immediately following a spinal cord injury (SCI). Which is an acute complication of SCI?

Spinal shock

A client sustains an injury to the ligaments surrounding a joint. The nurse identifies this as which of the following?

Sprain

Which term refers to an injury to ligaments and other soft tissues surrounding a joint?

Sprain

When assessing the client for scoliosis, the nurse:

Stands behind the client and asks the client to bend forward at the waist

A client is diagnosed with osteomyelitis. This is most commonly caused by which of the following?

Staphylococcus aureus

Which condition occurs when blood collects between the dura mater and arachnoid membrane?

Subdural hematoma

An adult is swinging a small child by the arms, and the child screams and grabs his left arm. It is determined in the emergency department that the radial head is partially dislocated. What is this partially dislocated radial head documented as?

Subluxation

The nurse is caring for a patient with a bone tumor. The nurse provides education that teaches the patient to implement measures to reduce the risk of pathologic fractures. What intervention will assist the patient in fracture prevention?

Supporting the affected extremity with external supports (splints)

Which of the following provides the best outcome for most tumor types?

Surgery

A client with chronic osteomyelitis has undergone 6 weeks of antibiotic therapy. The wound appearance has not improved. What action would the nurse anticipate to promote healing?

Surgical debridement

Morton neuroma is exhibited by which clinical manifestation?

Swelling of the third (lateral) branch of the median plantar nerve

Skull sutures are an example of which type of joint?

Synarthrosis

Autonomic dysreflexia can occur with spinal cord injuries above which of the following levels?

T6

Which of the following is not a manifestation of Cushing's Triad?

Tachycardia

The nurse received the report from a previous shift. One of her clients was reported to have a history of basilar skull fracture with otorrhea. What assessment finding does the nurse anticipate?

The client has cerebral spinal fluid (CSF) leaking from the ear.

A patient's recent diagnostic workup has resulted in a diagnosis of a glioma, and a treatment plan is being promptly created by the multidisciplinary care team. The patient's oncologist has recommended chemotherapy, which is to be administered by the intrathecal route. The nurse should understand that the rationale for choosing this administration route involves which of the following considerations?

The drug will bypass the blood-brain barrier.

A 14-year-old boy was brought to the emergency department (ED) by his father after suffering an apparent concussion during a game. Assessment in the ED confirmed the father's suspicion, and the boy is being discharged home in his father's care. What health education should the nurse provide to the boy's father?

The father should awaken his son every 2 hours during the night.

The emergency room nurse is reporting the location of a fracture to the client's primary care physician. When stating the location of the fracture on the long shaft of the femur, the nurse would be most correct to state which terminology locating the fractured site?

The fracture is on the diaphysis.

After several weeks of antibiotic therapy for the treatment of osteomyelitis, a patient is preparing for discharge. When providing health education related to self-care, the nurse should emphasize which of the following topics?

The importance of adhering to further antibiotic treatment

A nurse is caring for a client who's experiencing septic arthritis. This client has a history of immunosuppressive therapy and his immune system is currently depressed. Which assignment is the most appropriate for the nurse caring for this client?

The nurse is caring for this client on the intensive care unit.

Which statement(s) reflect nursing interventions for a client with post-polio syndrome?

The nurse provides care aimed at slowing the loss of strength and maintaining the physical, psychological and social well being of the client

A patient has an S5 spinal fracture from a fall. What type of assistive device will this patient require?

The patient will be able to ambulate independently.

A 77-year-old man is recovering in the hospital after a recent femoral fracture and has rung his call light. The nurse has entered the room to find the patient in distress, clutching his chest while struggling to say, "I can't breathe." The nurse should take prompt action based on the knowledge that this patient may be experiencing what complication of lower extremity fractures?

Thromboembolism

A nurse is teaching a community class that those experiencing symptoms of ischemic stroke need to enter the medical system early. The primary reason for this is which of the following?

Thrombolytic therapy has a time window of only 3 hours.

The nursing is assessing a client who has been diagnosed with a pituitary adenoma, but has not yet started treatment. The client reports having increased heart rate, hand tremors, difficulty sleeping, weight loss and hyperthermia. The nurse anticipates the client will require blood work to assess for overproduction of which hormone?

Thyroid-stimulating hormone

Which sign may be helpful in identifying carpal tunnel syndrome?

Tinel's Sign percuss lightly over median nerve (located on inner aspect of wrist). If pt reports pain, numbness and tingling, Test is positive

The nurse is seeing a client who is suspected of having a glioblastoma multiforme tumor. The nurse anticipates the client will require which diagnostic test to confirm the client has this form of brain tumor?

Tissue biopsy

A client is hospitalized when presenting to the emergency department with right-sided weakness. Within 6 hours of being admitted, the neurologic deficits had resolved and the client was back to his presymptomatic state. The nurse caring for the client knows that the probable cause of the neurologic deficit was what?

Transient ischemic attack

Which of the following are associated with compartment syndrome? Select all that apply.

Trauma from accidents Surgery Casts Tight bandages Crushing injuries

A client is diagnosed with meningococcal meningitis. The 22-year-old client shares an apartment with one other person. What would the nurse expect as appropriate care for the client's roommate?

Treatment with antimicrobial prophylaxis as soon as possible

A 45-year-old client is admitted to the facility with excruciating paroxysmal facial pain. He reports that the episodes occur most often after feeling cold drafts and drinking cold beverages. Based on these findings, the nurse determines that the client is most likely suffering from which neurologic disorder?

Trigeminal neuralgia

A patient with a history of chronic foot ulcers secondary to diabetes has been admitted to the preoperative clinic in preparation for a metatarsal amputation. The patient appears stoic and avoids making eye contact with the nurse, while answering assessment questions with one-word answers. How should the nurse best respond to this patient's demeanor?

Try to gently assess the patient's feelings around this procedure.

The nurse is assessing the client's pupils following a sports injury. Which of the following assessment findings indicates a neurologic concern? Select all that apply.

Unequal pupils Pinpoint pupils Absence of pupillary response

A nurse is caring for a client who underwent a lumbar laminectomy 2 days ago. Which finding requires immediate intervention?

Urine retention or incontinence

A high school soccer player sustained five concussions before she was told that she should never play contact sports again. After her last injury, she began experiencing episodes of double vision. She was told that she had most likely incurred damage to which cranial nerve?

VI (Abducens)

The nurse who is employed in a neurologist's office is performing a history and assessment on a client experiencing hearing difficulty. The nurse is most correct to gather equipment to assess the function of cranial nerve:

VIII

A patient has been brought to the emergency department (ED) with signs and symptoms of a stroke and a stat computed tomography (CT) head scan has been ordered. The ED nurse should know that the image that results from CT indicates distinguishing differences based on which of the following variables?

Variations in tissue density

A client has sustained a head injury to the occipital area. He cannot identify a familiar object by looking at it. The nurse knows that this deficit is which of the following?

Visual agnosia

A client is recovering from a fractured hip. The nurse would suggest that the client increase intake of which of the following to facilitate calcium absorption from food and supplements?

Vitamin D

The nurse teaches the client with a high risk for osteoporosis about risk-lowering strategies, including which action?

Walk or perform weight-bearing exercises outdoors

A nurse is planning discharge teaching regarding exercise for a client at risk for osteoporosis. Which of the following exercises would be appropriate?

Walking

The nurse is providing information about strokes to a community group. Which of the following would the nurse identify as the primary initial symptoms of an ischemic stroke?

Weakness on one side of the body and difficulty with speech

A client is scheduled for standard EEG testing to evaluate a possible seizure disorder. Which nursing intervention should the nurse perform before the procedure?

Withhold anticonvulsant medications for 24 to 48 hours before the exam

The nurse educator is providing orientation to a new group of staff nurses on an oncology unit. Part of the orientation is to help nurses understand the differences between various types of brain tumors. The nurse educator correctly identifies that glioma tumors are classified based on the fact that they originate where in the brain?

Within the brain tissue

A patient has just had an arthroscopy. What is a nursing intervention that is necessary for the nurse to implement following an arthroscopy?

Wrap the joint in compression dressing.

Which are risk factors for spinal cord injury (SCI)? Select all that apply.

Young age Alcohol use Drug abuse

While reading a client's chart, the nurse notices that the client is documented to have paresthesia. The nurse plans care for a client with

abnormal sensations.

A nurse is working in a neurologist's office. The physician orders a Romberg test. The nurse should have the client:

close his or her eyes and stand erect

Lower motor neuron lesions cause

flaccid muscles

A client is being admitted to a rehabilitation hospital as a result of the tetraplegia caused a stroke. The client's condition is stable, and after admission the client will begin physical and psychological therapy. An important part of nursing management is to reposition the client every 2 hours. What is the rationale behind this intervention?

maintain sufficient integument capillary pressure

The nurse is caring for a comatose client. The nurse knows she should assess the client's motor response. Which method may the nurse use to assess the motor response?

observing the client's response to painful stimulus

A nurse is completing a neurological assessment and determines that the client has significant visual deficits. Considering the functions of the lobes of the brain, which area will most likely contain the neurologic deficit?

occipital

A fracture is considered pathologic when it

occurs through an area of diseased bone.

The nurse is educating a client with low back pain on proper lifting techniques. The nurse recognizes that the education was effective when the client

places the load close to the body.

The nurse is conducting an admission history of a client admitted with a fracture. The nurse recognizes that which of the client's medications placed the client at risk for fractures?

prednisone (Deltasone)

A client's spouse relates how the client reported a severe headache, and shortly after was unable to talk or move their right arm and leg. The spouse indicates the client has hypertension. What should be the focus of management during this phase?

preventing further neurologic damage

A client who has been severely beaten is admitted to the emergency department. The nurse suspects a basilar skull fracture after assessing:

raccoon's eyes and Battle sign.

The nurse teaches the client that corticosteroids will be used to treat his brain tumor to

reduce cerebral edema.

A nurse is caring for a client with an injury to the central nervous system. When caring for a client with a spinal cord insult slowing transmission of the motor neurons, the nurse would anticipate a delayed reaction in:

response due to interrupted impulses from the central nervous system

A nurse is teaching a client with multiple sclerosis (MS). When teaching the client how to reduce fatigue, the nurse should tell the client to:

rest in an air-conditioned room.

A client admitted to the emergency department is being evaluated for the possibility of a stroke. Which assessment finding would lead the nurse to suspect that the client is experiencing a hemorrhagic stroke?

severe exploding headache

Which are characteristics of autonomic dysreflexia?

severe hypertension, slow heart rate, pounding headache, sweating

A nurse in a rehabilitation facility is coordinating the discharge of a client who is tetraplegic. The client, who is married and has two children in high school, is being discharged to home and will require much assistance. Who would the discharge planner recognize as being the most important member of this client's care team?

spouse

An example of a flat bone is the

sternum

A client has been diagnosed with a muscle strain. What does the physician mean with the term "strain"?

stretched or pulled beyond its capacity

During a routine physical examination to assess a client's deep tendon reflexes, a nurse should make sure to:

support the joint where the tendon is being tested.

The nurse is instructing a community class when a student asks, "How does someone get super strength in an emergency?" The nurse should respond by describing the action of the:

sympathetic nervous system.

The Monro-Kellie hypothesis explains

the dynamic equilibrium of cranial contents.

During a routine physical examination on a 75-year-old female client, a nurse notes that the client is 5 feet, 3/8 inches (1.6 m) tall. The client states, "How is that possible? I was always 5 feet and 1/2? (1.7 m) tall." Which statement is the best response by the nurse?

"After menopause, the body's bone density declines, resulting in a gradual loss of height."

A client preparing to undergo a lumbar puncture states he doesn't think he will be able to get comfortable with his knees drawn up to his abdomen and his chin touching his chest. He asks if he can lie on his left side. Which statement is the best response by the nurse?

"Although the required position may not be comfortable, it will make the procedure safer and easier to perform."

The nurse is providing end-of-life care to a client who was diagnosed with glioblastoma multiforme (GBM) 8 months ago. Despite a calm interaction with the client 1 hour ago, the client is now angry and yells, "Get out of my room and don't touch me anymore. I don't need your help!" How should the nurse respond?

"I can tell now is not the right time for me to come in and check on you. Please let me know when it is a better time for me to come back."

The client has just been diagnosed with osteomyelitis. Osteomyelitis is an infection of the bone, resulting in limited blood supply to the bone, inflammation of and pressure on the tissue, bone necrosis, and formation of new bone around devitalized bone tissue. What are possible causes of osteomyelitis? Select all that apply.

-Trauma, such as penetrating wounds or compound fractures -Vascular insufficiency in clients with diabetes or peripheral vascular disease -Surgical contamination, such as pin sites of skeletal traction

A nurse knows that, for a patient with an ischemic stroke, tPA is contraindicated if the blood pressure reading is:

190 mm Hg/120 mm Hg

Which client would the nurse identify as having the greatest risk for osteoporosis?

A small-framed, thin 45-year-old white woman

The nurse in the emergency department is caring for a patient brought in by the rescue squad after falling from a second-story window. The nurse assesses ecchymosis over the mastoid and clear fluid from the ears. What type of skull fracture is this indicative of?

Basilar skull fracture

Which of the following diagnostic studies provides visualization of cerebral blood vessels?

Cerebral angiography

Which occurs when reflexes are hyperactive when the foot is abruptly dorsiflexed?

Clonus

Which posture exhibited by abnormal flexion of the upper extremities and extension of the lower extremities?

Decorticate

Which of the following clinical manifestations would the nurse expect to find in a client who has had osteoporosis for several years?

Decreased height

A nurse working on a medical-surgical floor walks into a patient's room to find the patient with an altered level of consciousness (LOC). Which of the following nursing diagnoses would be the first priority for the plan of care?

Ineffective airway clearance related to altered LOC

A 71-year-old man has made an appointment with his primary care provider at the urging of his wife, who states that he has occasionally had episodes of weakness and slurring of words over the past several weeks. The care provider recognizes the possibility that the man has been experiencing transient ischemic attacks (TIAs). TIAs have which of the following characteristics?

TIAs cause symptoms that last less than 1 hour.

The nurse is preparing discharge teaching for a patient who is being discharged after hospitalization for a hemorrhagic stroke. What should be included in the discharge teaching for this patient?

Take antihypertensive medication as ordered.

A client experiences a musculoskeletal injury that involves the structure that connects a muscle to the bone. The nurse understands that this injury involves which of the following?

Tendon

Which medication taken by the client in the previous 24 hours would be of greatest concern to the nurse caring for a client undergoing a bone biopsy?

aspirin

If a client has a lower motor neuron lesion, the nurse would expect the client to exhibit

decreased muscle tone

A nurse is monitoring a client for increasing intracranial pressure (ICP). Early signs of increased ICP include:

diminished responsiveness.

A client who has fallen and injured a hip cannot place weight on the leg and is in significant pain. After radiographs indicate intact but malpositioned bones, what would the physician diagnose?

dislocation

The type of fracture described as having one side of the bone broken and the other side bent would be:

greenstick

A nurse suspects that a client with a recent fracture has compartment syndrome. Assessment findings may include:

inability to perform active movement and pain with passive movement

A client has an exaggerated convex curvature of the thoracic spine. What is this condition called?

kyphosis

A client has had surgical repair of a hip injury after joint manipulation was unsuccessful. After surgery, the nurse implements measures to prevent complications. Which complications is the nurse seeking to prevent? Select all that apply.

skin breakdown wound infection pneumonia

The nurse is providing care for a client who just discussed palliative care with the primary health care provider. The client's family member was also part of the discussion and asks the nurse, "I feel like this kind of treatment means we've given up on trying." How should the nurse respond? Choose the best option.

"The goal of this type of care is to promote the best quality of life that is now possible."

When caring for a patient who has had a hemorrhagic stroke, close monitoring of vital signs and neurological status is imperative. What is the earliest sign of deterioration in a patient with a hemorrhagic stroke?

Alteration in level of consciousness (LOC)

The ED nurse is receiving a client handoff report at the beginning of the nursing shift. The departing nurse notes that the client with a head injury shows Battle sign. The incoming nurse expects which to observe clinical manifestation?

An area of bruising over the mastoid bone

A nurse who provides care at a community clinic is in contact with a diverse group of patients. Which of the following individuals most clearly displays risk factors for stroke?

An obese woman with a history of atrial fibrillation and type 2 diabetes

A patient has been admitted to a unit at a primary stroke center after experiencing an ischemic stroke. The nurse on the unit is aware of the vital importance of rehabilitative efforts and knows that an active rehabilitation program should begin at what point?

As soon as the patient regains consciousness

Which positions is used to help reduce intracranial pressure (ICP)?

Avoiding flexion of the neck with use of a cervical collar

At a certain point, the brain's ability to autoregulate becomes ineffective and decompensation (ischemia and infarction) begins. Which of the following are associated with Cushing's triad? Select all that apply.

Bradycardia Hypertension Bradypnea

At which of the following spinal cord injury levels does the patient have full head and neck control?

C5

A female patient tells the nurse that she has pain and numbness to her thumb, first, and second finger of her right hand. The nurse discovers that the patient is employed as an auto mechanic, and that the pain is increased while working. This may indicate that the patient could possibly have what?

Carpal tunnel syndrome

A client comes to the clinic for evaluation because of complaints of dizziness and difficulty walking. Further assessment reveals a staggering gait, marked muscle incoordination, and nystagmus. A brain tumor is suspected. Based on the client's assessment findings, the nurse would suspect that the tumor is located in which area of the brain?

Cerebellum

A patient presents to the emergency room with complaints of having an "exploding headache" for the last 2 hours. The patient is immediately seen by a triage nurse who suspects the patient is experiencing a stroke. Which of the following is a possible cause based on the characteristic symptom?

Cerebral aneurysm

Corticosteroids are used in the treatment of brain tumors for which of the following clinical manifestations? Select all that apply.

Cerebral edema Headache Altered level of consciousness

An x-ray demonstrates a fracture in which a bone has splintered into several pieces. Which type of fracture is this?

Comminuted

Two days after application of a cast to treat a fractured femur, the patient tells the orthopedic health care provider that he is experiencing severe, deep, and constant pain in his leg. The nurse suspects a diagnosis of:

Compartment syndrome

A 58-year-old construction worker fell from a 25-foot scaffolding and incurred a closed head injury as a result. As his intracranial pressure continues to increase, the potential of herniation also increases. If the brain herniates, which of the following are potential consequences? Choose all correct options.

Death Permanent neurologic dysfunction Impaired cellular activity

A community health nurse is participating in a health forum under the auspices of the local seniors' center. The subject of osteoporosis has come up during a discussion with participants, and the nurse is providing a brief overview of the concept of bone density and the changes in bone density that occur with age. What age-related change in hormone levels contributes to the high incidence and prevalence of osteoporosis among older adults?

Decreased levels of calcitonin

Which diagnostic test does the nurse expect the client with osteoporosis to undergo?

Dual-energy x-ray absorptiometry

While doing an initial assessment on a patient, the patient tells the nurse that she has bone pain. The nurse asks the patient to describe the characteristics of the pain. Which of the following are typical characteristics of bone pain?

Dull, deep ache

A patient is being tested for a gag reflex. When the nurse places the tongue blade to the back of the throat, there is no response elicited. What dysfunction does the nurse determine the patient has?

Dysfunction of the vagus nerve

The nurse is performing an assessment for a patient in the clinic with Parkinson's disease. The nurse determines that the patient's voice has changed since the last visit and is now more difficult to understand. How should the nurse document this finding?

Dysphonia

A client arrives at the emergency department complaining of extreme muscle weakness after minimal effort. The physician suspects myasthenia gravis. Which drug will be used to test for this disease?

Edrophonium (Tensilon)

A client is undergoing testing to confirm a diagnosis of myasthenia gravis. The nurse explains that a diagnosis is made if muscle function improves after the client receives an IV injection of a medication. What is the medication the nurse tells the client he'll receive during this test?

Edrophonium (Tensilon)

An acoustic neuroma is a benign tumor of which cranial nerve?

Eighth

A patient is diagnosed with a fracture of a diarthrosis joint. The nurse knows that an example of this type of joint is the:

Elbow.

A client has delayed bone healing in a fractured right humerus. What should the nurse prepare the client for that promotes bone growth?

Electrical stimulation

The nurse would expect which of the following diagnostic tests to be ordered for a patient with lower extremity muscle weakness?

Electromyograph (EMG)

Which term refers to a method of recording, in graphic form, the electrical activity of a muscle?

Electromyography

A 53-year-old man presents to the emergency department with a chief complaint of inability to form words, and numbness and weakness of the right arm and leg. Where would you locate the site of injury?

Left frontoparietal region

Which of the following is accurate regarding a hemorrhagic stroke?

Main presenting symptom is an "exploding headache."

A client is exhibiting signs of increasing intracranial pressure (ICP). Which intravenous solution (IV) would the nurse anticipate hanging?

Mannitol

Osmotic diuretics are an essential intervention for reducing cerebral edema. Which of the following drugs is most frequently prescribed for this situation?

Mannitol

The nurse explains to the client with projectile vomiting and severe headache that a medication is being prescribed to reduced edema surrounding the brain and lessen these symptoms. What medication is the nurse preparing to administer?

Mannitol

An emergency department nurse has just received a call from EMS that they are transporting a 17-year-old male who has just sustained a spinal cord injury (SCI). The nurse recognizes that the most common cause of this type of injury is what?

Motor vehicle accidents

Which is a chronic, degenerative, progressive disease of the central nervous system characterized by the occurrence of demyelination in the brain and spinal cord?

Multiple sclerosis

The nurse is caring for a client with a traumatic brain injury who has developed increased intracranial pressure resulting in syndrome of inappropriate antidiuretic hormone (SIADH). While assessing this client, the nurse expects which of the following findings?

Oliguria and serum hyponatremia

An emergency department nurse is assessing an 80-year-old patient who has presented with a complaint of abdominal pain. The nurse performs a rapid inspection of the patient and notes multiple significant findings, including kyphosis. The nurse should understand that this assessment finding is suggestive of what musculoskeletal disease?

Osteoporosis

The most common cause of cholinergic crisis includes which of the following?

Overmedication

The nurse is performing an assessment for a patient who may have peripheral neurovascular dysfunction. What signs does the patient present with that indicate circulation is impaired? (Select all that apply.)

Pale, cyanotic, or mottled color Cool temperature of the extremity More than 3-second capillary refill

Which term is used to describe edema of the optic nerve?

Papilledema

The nurse educator is facilitating a class on neurological function with a group of nursing students. When discussing problems that can result from growing brain tumors, the nurse educator should include that clients can experience which neurologic deficits even after surgical resection? Select all that apply.

Paralysis Incontinence Aphasia

A 30-year-old female patient has been referred to a specialist by her primary care provider because she has recently developed fat pads at the base of her neck, an increasingly round face, and striae at various locations. The patient's signs of illness have been attributed to a brain tumor. What type of brain tumor is most likely to result in these changes to the woman's physical appearance?

Pituitary adenoma

The nurse educator is teaching nursing students about various types of brain tumors. The instructor recognizes that teaching has been effective when students correctly identify a client whose lab work indicates excessively high levels of thyroid stimulating hormone would most likely be diagnosed with which type of tumor?

Pituitary adenoma

A hospital patient's most recent blood work reveals a serum calcium level of 6.9 mg/dL (normal 8.5 to 0.5 mg/dL). In response to this low level of calcium, what physiological response is most likely to occur?

Secretion of parathormone

A client has a herniated disk in the region of the third and fourth lumbar vertebrae. Which nursing assessment finding most supports this diagnosis?

Severe lower back pain

A nurse practitioner assesses a patient's movement in his left hand after a cast is removed. The nurse asks the patient to turn his wrist so the palm of his hand is facing up. This movement is known as:

Supination.

A client has sustained a traumatic brain injury with involvement of the hypothalamus. The nurse is concerned about the development of diabetes insipidus. Which of the following would be an appropriate nursing intervention to monitor for early signs of diabetes insipidus?

Take daily weights.

The nurse working in the orthopedic surgeon's office is asked to schedule a shoulder arthrography. The nurse determines that the surgeon suspects which finding?

Tear in the joint capsule

The nurse is caring for a client with traumatic brain injury (TBI). Which clinical finding, observed during the reassessment of the client, causes the nurse the most concern?

Temperature increase from 98.0°F to 99.6°F

A client has been treated for migraine headaches for several months and comes to the clinic stating he is getting no better. The nurse is talking with the client and hears an audible click when the client is moving his jaw. What does the nurse suspect may be happening?

Temporomandibular disorder

Which client should the nurse assess for degenerative neurologic symptoms?

The client with Huntington disease.

The nurse is providing care for a male patient who has undergone knee arthroplasty. As part of the nurse's morning assessment, the nurse is assessing for peripheral neurovascular dysfunction distal to the surgical site. When performing this assessment, what parameters should the nurse assess and document? Select all that apply.

The color of the patient's lower leg and foot The patient's ability to move his foot The patient's sensation in his foot and lower leg The temperature of the patient's foot and lower leg

Identify descriptors of the pathophysiologic process seen in osteomalacia. Select all that apply.

There is a deficiency of activated vitamin D (calcitriol). Calcium and phosphate are not moved to the bones. The bone mass is structurally weaker, and bone deformities occur.

The nurse is seeing the mother of a client who states, "I'm so relieved because my son's doctor told me his brain tumor is benign." The nurse knows what is true about benign brain tumors?

They can affect vital functioning.

An infusion of phenytoin (Dilantin) has been ordered for a patient whose brain tumor has just caused a seizure. The patient has been receiving D5W at 100 mL/hour to this point and has only one IV access site at this point. How should the nurse prepare to administer this drug to the patient?

Thoroughly flush the patient's IV with normal saline.

A nurse provides nutritional health teaching to a 52-year-old who had two fractures in 1 year. Besides recommending supplemental calcium, the nurse suggests a high-calcium diet. The nurse would recommend that the patient increase her intake of:

Yogurt and cheese.

A client was hit in the head with a ball and knocked unconscious. Upon arrival at the emergency department and subsequent diagnostic tests, it was determined that the client suffered a subdural hematoma. The client is becoming increasingly symptomatic. How would the nurse expect this subdural hematoma to be classified?

acute

The nurse is performing a musculoskeletal assessment of a client in a nursing home who had a stroke 2 years ago and who has right-sided hemiplegia. The nurse notes that the girth of the client's right calf is 2 inches less in diameter than the left calf. The nurse attributes the decreased girth to

atrophy of right calf muscle

There are a variety of problems that can become complications after a fracture. Which is described as a condition that occurs from interruption of the blood supply to the fracture fragments after which the bone tissue dies, most commonly in the femoral head?

avascular necrosis

A nurse is caring for a client who has a history of a cerebral aneurysm. Which diagnostic test does the nurse anticipate to monitor the status of the aneurysm?

cerebral angiography

While snowboarding, a fell and sustained a blow to the head, resulting in a loss of consciousness. The client regained consciousness within an hour after arrival at the ED, was admitted for 24-hour observation, and was discharged without neurologic impairment. What would the nurse expect this client's diagnosis to be?

concussion

Medical management of arthropod-borne virus (arboviral) encephalitis is aimed at

controlling seizures and increased intracranial pressure.

A client is brought to the emergency department after being struck with a baseball bat on the upper arm while diving for a pitched ball. Diagnostic tests reveal that the humerus is not broken but that the client has suffered another type of injury. What type of injury would the physician likely diagnose?

contusion

A client is waiting in a triage area to learn the medical status of family members following a motor vehicle accident. The client is pacing, taking deep breaths, and handwringing. Considering the effects in the body systems, the nurse anticipates that the liver will:

convert glycogen to glucose for immediate use

A client has a fractured femur and is being seen in the emergency department. The nurse assessing the area notices there is a grating sound that is suspected to be bone ends moving over one another. This would be called:

crepitus

The initial sign of increasing intracranial pressure (ICP) includes

decreased level of consciousness.

A client who has injured a hip in a fall cannot place weight on the leg and is in significant pain. After radiographs indicate intact yet malpositioned bones, what repair would the physician to perform?

joint manipulation and immobilization

A nurse is working on a surgical floor. The nurse must logroll a client following a:

laminectomy

Bone density testing in clients with post-polio syndrome has demonstrated

low bone mass and osteoporosis.

Bursitis is an inflammation of the bursa, a fluid-filled sac that cushions bone ends to enhance a gliding movement. What possible assessment findings would be present in a client with bursitis? Select all that apply.

painful movement of a joint a distinct lump

The nurse has completed evaluating the client's cranial nerves. The nurse documents impairment of the right cervical nerves (CN IX and CN X). Based on these findings, the nurse should instruct the client to

refrain from eating or drinking for now.

A nurse is assessing a client who has been in a motor vehicle collision. The client directly and accurately answers questions. The nurse notes a contusion to the client's forehead; the client reports a headache. Assessing the client's pupils, what reaction would confirm increasing intracranial pressure?

unequal response

A client has been diagnosed with a concussion and is to be released from the emergency department. The nurse teaches the family or friends who will be caring for the client to contact the physician or return to the ED if the client

vomits


Kaugnay na mga set ng pag-aaral

Chapter 2 Barron's AP Psychology

View Set

Computer Organization homework one

View Set

SOCIAL LEARNING THEORY - ALBERT BANDURA

View Set

Account 101- Ch 2: Accounting Cycle

View Set

Chapter 4 Nucleic Acids and RNA World.

View Set